Difference between revisions of "2021 AMC 10B Problems/Problem 17"

m (Solution 3 (Comprehensive but Unnecessary))
m (Solution 3 (Comprehensive but Unnecessary): Shortened some word choice.)
Line 25: Line 25:
  
 
==Solution 3 (Comprehensive but Unnecessary)==
 
==Solution 3 (Comprehensive but Unnecessary)==
Using observations, we consider the scores from lowest to highest. We make the following logical deduction:
+
By observations, we consider the scores from lowest to highest. We make the following logical deduction:
  
 
<cmath>\begin{align*}
 
<cmath>\begin{align*}
Line 37: Line 37:
 
Therefore, the answer is <math>\boxed{\textbf{(C) }\text{Ravon was given card 4.}}</math>
 
Therefore, the answer is <math>\boxed{\textbf{(C) }\text{Ravon was given card 4.}}</math>
  
Of course, if we look at the answer choices earlier, then we can stop at <math>(*)</math> and pick <math>\textbf{(C)}.</math>
+
Of course, if we read the answer choices sooner, then we can stop at <math>(*)</math> and pick <math>\textbf{(C)}.</math>
  
 
~MRENTHUSIASM
 
~MRENTHUSIASM

Revision as of 03:28, 2 April 2021

Problem

Ravon, Oscar, Aditi, Tyrone, and Kim play a card game. Each person is given $2$ cards out of a set of $10$ cards numbered $1,2,3, \dots,10.$ The score of a player is the sum of the numbers of their cards. The scores of the players are as follows: Ravon--$11,$ Oscar--$4,$ Aditi--$7,$ Tyrone--$16,$ Kim--$17.$ Which of the following statements is true?

$\textbf{(A) }\text{Ravon was given card 3.}$

$\textbf{(B) }\text{Aditi was given card 3.}$

$\textbf{(C) }\text{Ravon was given card 4.}$

$\textbf{(D) }\text{Aditi was given card 4.}$

$\textbf{(E) }\text{Tyrone was given card 7.}$

Solution 1

Oscar must be given 3 and 1, so we rule out $\textbf{(A) \ }$ and $\textbf{(B) \ }$. If Tyrone had card 7, then he would also have card 9, and then Kim must have 10 and 7 so we rule out $\textbf{(E) \ }$. If Aditi was given card 4, then she would have card 3, which Oscar already had. So the answer is $\boxed{ \textbf{(C) }\text{Ravon was given card 4.}}$

~smarty101 and smartypantsno_3

Solution 2

Oscar must be given 3 and 1. Aditi cannot be given 3 or 1, so she must have 2 and 5. Similarly, Ravon cannot be given 1, 2, 3, or 5, so he must have 4 and 7, and the answer is $\boxed{ \textbf{(C) }\text{Ravon was given card 4.}}$.

-SmileKat32

Solution 3 (Comprehensive but Unnecessary)

By observations, we consider the scores from lowest to highest. We make the following logical deduction:

\begin{align*} \text{Oscar's score is 4.} &\Longrightarrow \text{Oscar was given cards 1 and 3.} \\ &\Longrightarrow \text{Aditi was given cards 2 and 5.} \\ &\Longrightarrow \text{Ravon was given cards 4 and 7.} \hspace{13mm} (*) \\ &\Longrightarrow \text{Tyrone was given cards 6 and 10.} \\ &\Longrightarrow \text{Kim was given cards 8 and 9.} \end{align*}

Therefore, the answer is $\boxed{\textbf{(C) }\text{Ravon was given card 4.}}$

Of course, if we read the answer choices sooner, then we can stop at $(*)$ and pick $\textbf{(C)}.$

~MRENTHUSIASM

Video Solution by OmegaLearn (Using logical deduction)

https://youtu.be/zO0EuKPXuT0

~ pi_is_3.14

Video Solution by TheBeautyofMath

https://youtu.be/FV9AnyERgJQ?t=284

~IceMatrix

See Also

2021 AMC 10B (ProblemsAnswer KeyResources)
Preceded by
Problem 16
Followed by
Problem 18
1 2 3 4 5 6 7 8 9 10 11 12 13 14 15 16 17 18 19 20 21 22 23 24 25
All AMC 10 Problems and Solutions

The problems on this page are copyrighted by the Mathematical Association of America's American Mathematics Competitions. AMC logo.png